User Avatar
Benjamin Kieta
Instructor
Joined
Mar 2025
Benjamin is an experienced teacher with a background in language, philosophy, and rhetoric. His approach to teaching focuses on breaking complex problems into manageable pieces. He believes that success on the LSAT is based on developing strategies and thought structures that help a student read between the lines of what each question is asking. Benjamin has had the opportunity to study seven different languages and has lived in Colombia, the United States, China, Canada, and Hungary. Besides tutoring for the LSAT, he teaches English and classical Hebrew and works semi-professionally as a writer. In his free time, Benjamin enjoys playing violin and roasting coffee.
User Avatar
Benjamin Kieta
Thursday, Oct 30

Hi Kaylee!

Thanks for your question. Assumptions are far and away one of the most important topics on the LSAT, and we've got lots of questions about assumptions in general and each specific question type that incorporates an assumption. You can find a list of our upcoming classes on those topics here.

Looking forward to seeing you in class!

-Benjamin, Classes Operations Manager

PrepTests ·
PT130.S3.Q23
User Avatar
Benjamin Kieta
Wednesday, May 28

Student question:

Could (B) be considered a premise booster? We already know that, for the purposes of advancing our argument, the studies were not sufficiently representative (most studies didn’t include people with insomnia and for the few that did, only a few participants were significantly impacted). Telling us again that this study isn’t representative wouldn’t strengthen our argument.

Tutor answer:

At one level, I think I agree with you that it would essentially just be "boosting" the premise idea that the study isn't representative. On the other hand, I think it also continues the argument in another direction that is simply out of scope. In many cases, we care about whether or not a study is representative of the population as a whole. In this case, we don't; as a matter of fact, we're looking for a study that is relevant to one specific subset of the population, those with insomnia. I suppose I could even imagine a scenario where B would weaken the argument, if by being unrepresentative of the whole human population the studies were actually more focused on people with insomnia.

So as wrong answers go, I would probably classify this answer choice in the "irrelevant" box rather than the "premise booster" box, since it introduces a concern that simply isn't important to where we're trying to go.

PrepTests ·
PT107.S4.Q21
User Avatar
Benjamin Kieta
Wednesday, May 28

Student question:

If we were to draw a most statement and say that most weaklings are also cowards and that most cowards fail to be fools, would that also be a okay way to translate the conditional? Wouldn't the answer be the same?

Tutor answer:

You're asking a really important question about how to diagram this statement. The first thing that's important to note here is that "all too many" is not the same as "most".

If I'm ever trying to make sure that I'm translating a quantifier statement correctly, I like to try to use the same phrase with in a different situation. So let's try that here: "All too many apples are rotten." Does that mean most apples? I don't think so. I could imagine myself saying that if I just bought a batch of 20 apples and 7 of them are rotten. But it does mean that some apples are rotten. So here, "some" is the correct choice for the relationship.

That said, you are correct that in this case the conclusion would still be unsupported even if we accidentally translate that first relationship as a "most". It's really great that you're recognizing that this relationship:

A -m-> B -m-> C

does not imply that:

A <-s-> C.

Just to illustrate this point, you can try it with an obviously flawed argument:

Most astronauts who have visited the International Space Station are Americans, and most Americans have never been to outer space. Therefore, most astronauts who have visited the International Space Station have never been to outer space.

PrepTests ·
PT140.S1.Q16
User Avatar
Benjamin Kieta
Wednesday, May 28

Student question:

Is A a sufficient assumption here? And therefore would A have been correct if B was not an answer choice?

Tutor answer:

Answer choice (A) is a sufficient assumption that would make the conclusion true. That said, since this question is looking for a Necessary Assumption, the LSAT would never ask for an answer choice like (A), since it simply isn't required by the argument.

It's important to keep in mind that there won't ever be two answer choices that are both required for the arugment to work in an NA question, and in that sense there isn't ever really a "second best" answer choice. For NA questions, we're really not trying to strengthen the reasoning; we're just looking for something that the reasoning demands.

PrepTests ·
PT151.S2.Q14
User Avatar
Benjamin Kieta
Wednesday, May 28

Student question:

Could E strengthen the conclusion by affirming the idea that the people who were able to notice extroversion were not doing so because of prior knowledge? I understand E as saying that some of the faces were random).

Tutor answer:

I think you're right to focus specifically on the conclusion, and so let's really drill down on what that conclusion is saying. First off, notice that the conclusion is a causal claim. We're arguing why a phenomenon happens. Note also that it's the kind of conclusion that is affirming one idea at the expense of another idea—the explanation we like is primate biology, not culture alone. A good answer choice could strengthen the primate biology connection, undermine culture's sole claim, or do both at the same time.

Let's start by looking at the issue with (E). You're right that (E) seems to be saying that the people in the study were looking at a randomized human face—so not an actual person. This might be helpful if our conclusion were trying to downplay knowledge of an individual or actual person as a causal factor. But it's hard to see how it would help us rule out the influence of culture. We can examine it with this razor: Is it reasonable to conclude that if the cause for this phenomenon were solely cultural, then a person wouldn't be able recognize the disposition a composite image? I have to make some pretty big assumptions to get there.

(C), on the other hand, focuses on providing a causal mechanism for the concept of primate behavior. It points to a commonality, rooted in primate biology, that would result in the phenomena that we are describing. On strengthen questions, causal mechanisms are going to be your best friend—if we can explain why A causes B, that will strengthen the idea that it actually does.

PrepTests ·
PT135.S1.Q20
User Avatar
Benjamin Kieta
Wednesday, May 28

Student question:

Can you please explain why D is correct?

Tutor answer:

It's often a good idea to prephrase the flaw in the stimulus. To do that, we need to figure out what the conclusion is and why it doesn't follow from the premises offered. The conclusion here is:

"a small observational study must be more likely to have dramatic findings than a large randomized trial."

What is that based on? Two ideas:

  1. Newspapers generally report only on dramatic-sounding results.

  2. Newspapers more often report on small studies than large studies.

What's the flaw here? It's actually a well-disguised version of one of the LSAT's favorite flaws: the "fraction of what?" issue.

The conclusion flips the relationship. We know that a bigger fraction of newspaper stories are from small studies, but do we have information about the fraction of small studies that end up in newspapers?

Let's deconstruct that question by using an extreme example. Let's say that small studies are 100x more common than big studies. Even if big studies and small studies had the same chance of returning dramatic findings, we'd see a lot more dramatic results from small studies, and so we'd expect to find more small study results in newspapers.

And so, we need to know roughly how common small studies are relative to big studies. If there are significantly more small studies than big studies, that could be the reason we see more of their results in newspapers, not because they're more likely to be dramatic. That's what answer choice (D) says.

PrepTests ·
PT152.S1.Q19
User Avatar
Benjamin Kieta
Wednesday, May 21

Student Question:

JY mentions the word strength for D is a reason why it is the correct AC BUT I see some explanations eliminate A because of the "it is possible" but D reads similar with the "could" both present possibilities. so why is the “it is possible” an eliminator if the right AC is doing the same thing?

Tutor Answer:

You're right that both (A) and (D) introduce something that is simply possible—it could be true. The reason why (D) is better than (A) is because of what the answer choices say could be true.

In the case of answer choice (A), we're dealing with a possibility that could potentially compromise the experiment. If every one of the 500 families who was not given a medical self-help book did in fact go out and get one, that would certainly affect the outcome of the experiment. However, it starts to approach the issue of, "Is this critique reasonable?" We're dealing with a fairly large sample of people, and the results showed a significant discrepancy. Is it possible that literally every single one went out and got self-help books? Maybe, but that objection is unlikely enough that I would need to see some support for that idea before I could see it as a genuine flaw in the experiment. If nothing else, the size of the study alone suggests that this isn't very likely.

(D) is a good answer choice because it points to a fundamental logical issue in the argumentation. Based on the data provided, we actually have no good reason whatsoever to infer a causal relationship between better health and self-help books. X and Y can both cause Z; it's a mistake to think that we therefore know whether or not X causes Y.

So rather than the situation described in (A) which could theoretically happen (but isn't very likely), in this case, the "could" is talking about a situation which is much more neutral. We have absolutely no information about a causal relationship between these two factors, and the inference that there is a causal relationship points to a misunderstanding of the way that causation works.

PrepTests ·
PT106.S2.Q19
User Avatar
Benjamin Kieta
Wednesday, May 21

Student Question:

I understand, conflating, necessary and sufficient conditions in this question but I still think c is the right answer. Let's say that 51% of the mail is undamaged and correctly addressed and 41% of the mail is damaged and correctly addressed and it takes one day for the undamaged correctly address mail to arrive and three days for the correctly. This means that correctly addressed me I would take two days to arrive, which satisfies the first part of the stimulus. Now let's say that correctly addressed mail is 90% of the mail and incorrectly addressed mail is 10% of the maill and. damage mail takes 10 days to arrive that would satisfy the last stimulus point that it takes longer than three days for most mail to arrive.

Tutor Answer:

Remember that with a Must Be True question, we're looking for an answer choice that reaches a somewhat higher threshold than simply satisfying conditions laid out in the stimulus. The two scenarios that you described in your question would certainly satisfy the stimulus and would describe a situation where most mail that arrives within two days is correctly addressed, but there is at least one scenario in which that wouldn't be true.

Imagine that incorrectly addressed mail represents 90% of all mail, and correctly addressed mail only represents the other 10%. Let's say that 99.9% of correctly addressed mail arrives within two business days. However, 35% of incorrectly addressed mail arrives in two business days, too. In this scenario, about 55% of all mail arrives in three days or more, but the majority of mail that arrives within two business days would be incorrectly addressed.

The fundamental problem with (C) is that we know something about the percentage of correctly addressed mail that arrives in two days (almost all); but there's no way to gather information about what percentage of 2-day mail is correctly addressed. We can manipulate the correctly addressed mail's percentage of the total and the percentage of incorrectly addressed mail that arrives in two days to make correctly addressed mail a minority of 2-day mail.

PrepTests ·
PT140.S4.P3.Q19
User Avatar
Benjamin Kieta
Wednesday, Jun 11

Student question:

I don't understand why A is wrong. The passage said if it was typical of a chess game, they would have exceptional memory on it. So my thinking was if its not typical of a chess game (maybe some other game) then they would not have exceptional memory of it. I don't know how I was supposed to assume that typical meant "easy" since the right answer choice was C, that said they don’t have memory over challenging chess games.

Tutor answer:

Let's start with analyzing the type of question we're looking at here. This is a "stated" question, which means that it is looking for something that the passage explicitly says, not simply something that we might be able to infer from the passage.

Moving on, let's look at the specific line of the passage that the question is referencing as it says that chess players do not have exception memory for something. That idea comes from the end of the second paragraph: "superior chess players have memories for configurations of chess pieces, but only if those configurations are typical of chess games."

(A) is wrong because it talks about games other than chess. But note that this idea isn't in scope for what is being tested with the chess players—we hear that we are specifically looking at how their memory was for configurations of chess pieces, including those typical and atypical of a chess game. It's never stated in the passage how chess players may or may not have memory of a game other than chess, including another hypothetical game that would use chess pieces but is not chess. Because of that, (A) will fall short of the standard that we use for a correct answer in a Stated question.

Please also note that (C) is not the correct answer here, either. Your objection that we have no evidence for the concept of "easy" from the word "typical" is completely correct.

The correct answer to this question is (D), which talks about some kinds of arrangements of chess pieces. That is what was being described in the passage—arrangements of chess pieces that are not typical of a chess game.

PrepTests ·
PT137.S2.Q14
User Avatar
Benjamin Kieta
Wednesday, Jun 11

Student question:

I tried the negation test and went with C. If there WERE plays that the critic has not seen this year, then wouldn’t this destroy the argument?

Tutor answer:

For answer choice (C) to be a necessary assumption, there's another assumption that we have to make. That other assumption is that the critic is only able to make these comments if they read or saw all of the plays written last year performed.

In this case, we don't know that. How the critic is getting the information isn't part of the argument. In fact, the idea that "no plays examine human nature in a particularly skillful way" is offered to us as a premise. Since we're trying to find an assumption in the way that the critic moves from the assumptions to the conclusion, we simply have to take it for granted that the critic is correct about this assertion (whether from actually watching or reading the plays or from some other way of knowing about them).

To say it another way, we can't say based on the argument that the critic needs personal experience with each play in order to make this argument.

PrepTests ·
PT151.S3.Q10
User Avatar
Benjamin Kieta
Wednesday, Jun 11

Student question:

If I make the "without" into the "negate sufficient", and then I take the "no" as just a basic negation, I end up with:

Years of training → Should not prescribe medication.

Where am I going wrong?

Tutor answer:

Don't forget the "negate" part of the "negate sufficient"! Notice that if you do start with the "without" side, we end up with both the sufficient and the necessary being negated:

​ /years of training → /should prescribe medication

Which is equivalent to:

should prescribe medication → years of training

It looks to me like you may have forgotten the negation itself as you were writing this out—if you choose to do it this way, remember that you need to keep the negatives of both the "without" and the "no" when you translate it!

PrepTests ·
PT149.S3.Q5
User Avatar
Benjamin Kieta
Wednesday, Jun 11

Student question:

Does (C) weaken the argument or have no effect?

Tutor answer:

(C) weakens the argument. The stimulus states that India has a lower incidence of Alzheimer's than the world average, and it attributes this to the consumption of turmeric. However, (C) provides an alternative explanation—perhaps India simply has a lower than average percentage of its population at the age that typically develops Alzheimer's.

Imagine, for example, that I made the case to you that there must be something that kids eat at Meadow Hill Elementary School that prevents Alzheimer's, because the incidence there is much lower than the world average incidence of Alzheimer's. You would say that's a ridiculous argument; of course there is less Alzheimer's at a grade school, because the population there is much less at risk. That is essentially what answer choice (C) is doing.

PrepTests ·
PT111.S1.Q8
User Avatar
Benjamin Kieta
Wednesday, Jun 11

Student question:

How can I avoid getting baited by answer choices like (C)?

Tutor answer:

One line of attack for answer choices in a Weaken question is to ask, "Does this piece of evidence change anything?" In order for an answer choice to be correct on a Weaken question, we will need to find something that has an effect on the conclusion, in this case a value judgment ("the proposal is pointless") without discarding the premise ("hardly any lobsters live long enough to be harmed by those diseases").

Keeping the scope of the premise in mind can be helpful. We know that the diseases caused by pollution really don't cause much harm to the lobsters themselves. So if we're going to find an answer choice that "changes things," it has to give us a new idea that makes us think that the sewage does some kind of harm apart from the harm that the gill diseases do to the lobsters themselves. There's lots of angles that this could go (maybe the sewage causes a different harm to lobsters besides simply gill diseases), but fundamentally, the right answer needs to bring in some idea that "changes things" and suggests an additional way that harm is being done.

PrepTests ·
PT114.S4.Q13
User Avatar
Benjamin Kieta
Wednesday, Jun 11

Student question:

The first line of the passage is listed as a conclusion, but is it not providing context? It looks like the author is giving information from another source.

Tutor answer:

I really appreciate your question here, because it points to one of the really challenging things about approaching a stimulus. In some cases, the conclusion is really well-disguised without any indicators, and it looks just as well like it could be context.

On this question, it's actually impossible to know whether or not the first sentence is the conclusion or is context until we get through the entire argument. The reason we know that it has to be the conclusion is that there is nothing else in the argument that could possibly be a conclusion. Every other claim in the stimulus is simply a fact about the study and the data that was found in the study. On the LSAT, facts like these will always be premises—there's really no debating them.

Left without anything else that could be a conclusion, we have to return to that first sentence. Why is this a conclusion and not context? Because it takes the data from the study and makes a causal claim about it. We are no longer talking specifically about the facts of the study; we are now saying that those facts show that there is no causal relationship between advertising and a children's breakfast preferences. It's really well-disguised, but imagine I put the sentence at the end of the stimulus and re-worded it like this: "Therefore, this study shows that advertising does not cause significant changes in children's breakfast cereal preferences." It would be essentially the same claim—they're just trying to throw you off by making this claim look like context here.

PrepTests ·
PT104.S1.Q2
User Avatar
Benjamin Kieta
Wednesday, Jun 11

Student question:

Originally, when I mapped out the assumption in the argument I wrote /understand → /A or /B, so I thought that when taking its contrapositive, I would arrive at A and B → understand. I now know that the abstract form is /understand → /[A or B] (so that the contrapositive is A or B → understand). Why is that? What indicator in that conclusion sentence in the stimulus should I have made me known this?

Tutor answer:

Great question! And/or language is one of the most complicated conditional ideas on the LSAT, and unfortunately the ambiguities of the English language can make it even more frustrating. In this case, the issue is that this negation language FEELS as if it's presenting us with an "or" scenario, but it's actually giving us an "and" scenario.

Let's look really closely at the language here. The conclusion says, "So... 80 percent of the population does not know enough about medical concepts to make well-informed personal medical choices or to make good public policy decisions about health care."

The question to ask ourselves at this point is whether the language here is suggesting that 80% of the population either can't make good personal choices or can't make good policy decisions. Or is it saying that 80% is both unable to make good personal choices and unable to make good policy decisions?

Let's look at a parallel statement: "Until you turn 18, you will not be allowed to vote or purchase certain products." Does this mean that you will either be unable to vote or purchase certain products, or that you will both be unable to vote and purchase the products? It's the "both-and" one, right?

So to summarize, when we want to express an "and" negation in English, we often say, "Not A or B", which you can think of as /A and /B. Logically speaking, you can think of the negation as applying to the phrase "A or B"— /[A or B]. When we "distribute the negation", it turns into /A and /B.

PrepTests ·
PT143.S4.Q14
User Avatar
Benjamin Kieta
Monday, Jun 09

Student question:

I struggle with RRE questions, both with finding the phenomenon itself and then understanding what is a reasonable assumption to make on RRE questions. Why is (A) right and not (E)?

Tutor answer:

RRE questions are a great chance to practice the concept of a logic gap, which is found all over the LSAT. Bear in mind that RRE questions will have two ideas that feel like they contradict each other, and it's our job to explain how the two sides connect to each other. We're looking for the answer choice that tells the story of how these ideas can both be true.

The first step in working through an RRE is to ask, "What's strange about this information? What are the two ideas that I need to connect?" In this case, the two ideas are: 1. declining car thefts, and 2. thieves are more likely to be convicted.

In a way, this information isn't actually as paradoxical as RRE questions often are. We are simply looking for an answer choice that tells us why both of these things are happening. (A) does that. It gives a justification for lower automobile thefts (fewer thieves) and a reason those thieves are more likely to be caught (they don't abandon the cars as early). The problem with (E) is that it doesn't really explain why there are more convictions—in fact, it makes it seem like there should be less, since courts tend to be lenient with minors.

Using this question, let's talk about assumptions that will be legitimate and ones that won't. One way we can do this is by making a simple comparison between the two answer choices that seem attractive. Is it more likely that there are more convictions if people have not left the cars by the time the owners find out or because youths typically get more lenient sentences? One seems to move in the correct direction, while the other is at best ambiguous and possible tips in the opposite direction. This kind of comparison can help you feel a bit more confident when you're not sure about which assumptions are legitimate.

PrepTests ·
PT114.S4.Q14
User Avatar
Benjamin Kieta
Monday, Jun 09

Student question:

Is it a general rule that when you encounter a flaw question that has an answer choice that is referencing topics outside the scope of the stimulus that it is probably wrong? For example, in this question, answer choice B is referencing other possibilities outside of the scope of the environmental and safety benefits, so are those types of answers typically wrong?

Tutor answer:

I'm going to answer with a cautious "yes." A flaw question is always going to be looking for an issue that exists between the premises that are offered and the conclusion that is drawn from them. So in that sense, any answer choice that introduces an additional consideration is going to be irrelevant. We don't really care here about other benefits; the argument is that driving slowly does not have these particular benefits.

Flaw questions often have wrong answer choices that try to bait you into an additional consideration that isn't part of the argument. Always make sure you're clear about what conclusion claim the author is trying to make and why, and that can help you weed out irrelevant considerations.

That said, an additional consideration can be valuable when an argument makes a sweeping claim on the basis of a single consideration only. "Cake tastes good; therefore we should eat it for breakfast, lunch, and dinner." Note that the conclusion makes the logical error of assuming that this one consideration is decisive. In this situation, it would be appropriate to cite as a flaw the fact that the argument overlooks the possibility that cake may not be in the interest of best health.

So to summarize: the way to decide whether a consideration is meaningful is to pay close attention to the conclusion claim that the argument is trying to make.

PrepTests ·
PT115.S3.P3.Q20
User Avatar
Benjamin Kieta
Monday, Jun 09

Student question:

Does anyone have a suggestion of what made it obvious to them that that this passage explicitly blew up wave theory and that there was an entirely new way of seeing this? It’s frustrating because it made two questions (20, 21) that would’ve seemed much easier to me, much harder.

Tutor answer:

I'd like to start by saying that treating classical wave theory as if it were "blown up" is probably not the way that a scientist would describe the relationship between these two ideas, and so if you don't come away from the passage with that specific feeling, that's not necessarily a problem: it may be truth conceptually, but it's not the author's tone.

What is important to recognize is that the author makes it clear that classical wave theory ran into major obstacles that it was unable to account for—including one that it describes as a "catastrophe." Note also that the third paragraph tells us that Planck "broke with" wave theory in his ideas. That's really significant, because it means that the passage does not understand what Planck eventually developed as merely a development of wave theory but a genuinely new movement in the field.

Knowing that, I think it becomes a bit easier to approach questions 20 and 21. The author of the passage clearly feels that Planck's view was more accurate than what came before, and so answer choice (E) for question 20 fits well with the author's view of Planck's break with wave theory. Question 21 again depends on recognizing that these are two different things. We know that Planck did some work within the development of classical wave theory, but the passage only expands on the work he did that broke with it.

PrepTests ·
PT144.S2.Q11
User Avatar
Benjamin Kieta
Monday, Jun 09

Student question:

I don't get why (E) is wrong.

Tutor answer:

Let's start by getting really specific about the paradox that we're trying to solve in the stimulus. We have two products that usually sell approximately equally, and yet when we did two things to one of them that we would expect to make it sell more (putting it on sale and setting it near the front of the store), it actually sold less relative to the other product. Why did that happen?

The problem with (E) is that it simply doesn't address this paradox. Normally, people are buying these two hammers at approximately the same rate, even when they are at their normal price. Even if (E) is true, it does nothing to explain why the Maxlast hammers were suddenly getting fewer sales relative to the Styron hammers after they were discounted. Maybe people aren't making special trips for the Maxlast hammers, but we'd still expect them to at least sell at roughly the same rate as before the sale, right?

So (E) doesn't address why sales actually went down; at most it tells us why they didn't go up.

PrepTests ·
PT113.S3.Q21
User Avatar
Benjamin Kieta
Monday, Jun 09

Student question:

I found this question confusing. I haven’t learned about the belief vs facts flaw, is this something that will be covered further in the core curriculum? I’m also not sure how I am supposed to be able to answer this question in such a short amount of time.

Tutor answer:

This is a brutal Flaw question, and I totally hear your frustration with it! It's extremely rare to be asked to match both of the flaws committed by a stimulus. But the great news is that both of these flaws are fairly common, and so we can track them and be ready for them when they come up in the future.

I'd like to start by pointing your attention to the argument flaw cheat sheet in the Core Curriculum. This is your best one-stop guide to the different repeated flaws that you can expect to see on LSAT questions—and it's worth getting familiar enough with them that you can feel fairly confident you'll recognize them when you see them!

Second of all, I want to take a moment to address the belief vs. facts flaw, since I know it's especially tricky in this question. I'm going to draw an analogy that I think can make this more real.

A person who commits perjury in the state of Louisiana may receive a penalty of imprisonment up to 40 years. Let's say that Bob knows that Mark has committed perjury in Louisiana. Does that necessarily mean that Bob knows that Mark may receive up to 40 years in prison? Not necessarily. Because we don't know if Bob knows what the maximum penalty is for perjury in Louisiana (I certainly didn't until I looked it up a few minutes ago).

A similar issue exists here. Alicia may know that the bank deposit was made before 3 PM, but does she know anything about the relationship between the time of deposit and the time of crediting? Not necessarily. We don't know what Alicia knows about bank deposits.

PrepTests ·
PT104.S2.P4.Q25
User Avatar
Benjamin Kieta
Monday, Jun 09

Student question:

Why does the existence of other researchers who have used his tactics mean they are generally accepted? Couldn’t he have used the same tactics as two out of hundreds of researchers working on this topic? This feels like the kind of jump that we’ve been taught to avoid.

Tutor answer:

I appreciate your sensitivity to the potential for making an unreasonable assumption here! I think I would agree with the objection you're raising to (B) if the only piece of information we had to go on was the fact that we know that "other researchers" approached the question in essentially the same way. It's true—how do we know that this was enough to meet the threshold of "generally"?

Something that helps us here are some of the comments that are made in other paragraphs. Paragraph 2 calls Lowe's explanation "plausible." Paragraph 3 refers to some aspects of his theory as "our understanding of the collapse," which would seem to imply that experts have a relatively united view with Lowe on at least some of the foundational information.

When we add those pieces of evidence together, it seems that Lowe's approach is indeed "generally accepted."

PrepTests ·
PT139.S1.Q24
User Avatar
Benjamin Kieta
Wednesday, Jun 04

Student question:

Doesn’t AC E work because it suggests an alternative explanation that those that already don’t drink were likely to take the pledge, which reverses the causal relationship? I don’t get why the explanation says the two claims are logically equivalent.

Tutor explanation:

The important thing in understanding answer choice (E) is to recognize how the LSAT thinks about the word "many". The word "many" is very subjective, and when we're working with conditional diagramming, we often treat it as equivalent to the word "some." But in this question, "many" refers to the fact that there are, at least in the author's opinion, a significant number of people who report having taken the pledge and do not drink.

What's really important to keep in mind is that this is not a statement about percentage. It isn't a "most" statement, which would mean "more than 50%". For example, if I said that "many Americans live overseas", it could be less than 1% of the population, and yet still be millions of people. When I say that "many" people are something, it's really a comment about the number of people, not the percentage of a group. I could also legitimately say that many of the world's people who live overseas from their home country are Americans, since again, it could be millions of people.

So when I say that "many who took the pledge do not drink" it's also true that "many who do not drink took the pledge", since that number of people is the same number. It represents the overlap of those two groups, and in the author's perspective, that overlap constitutes "many" people.

On the other hand, I hear that "almost all" who drink did not take the pledge. That also means that almost none who drink did take the pledge.

So if there are "many" who took the pledge and don't drink, and "almost none" who took the pledge and do drink, I can legitimately say that there is a strong correlation between taking the pledge and not drinking, even though I'm dealing in very subjective terms.

For that reason, the issue here is with inferring a causal relationship where there is only a correlation, not with misunderstanding the logical implications of the concept of "many."

PrepTests ·
PT158.S4.Q21
User Avatar
Benjamin Kieta
Wednesday, Jun 04

Student question:

Just want to clarify the logic in answer choice D. “Only those” indicates a necessary condition here? So it would mean if you are required to pay a tax, then you will benefit from the results? And if you will not benefit from the results, you are not required to pay a tax?

Tutor answer:

Your interpretation of the implications of (D) is spot on! Saying "only those" would create a necessary condition for who should vote for a measure. But that doesn't justify the conclusion; in order to do that, we need to make benefit from the measure a sufficient condition for voting for it, which is what (E) does.

PrepTests ·
PT123.S2.Q14
User Avatar
Benjamin Kieta
Wednesday, Jun 04

Student question:

I am confused why E weakens the answer choice. If it is still the case that the microwaves are causing these small zones, then couldn’t we reasonably deduce that the microwaves still generate heat that destroys the enzyme by creating these small pockets?

Tutor answer:

The key to this question is to note very specifically the claim that the conclusion is trying to make. The person making the claim wants to make the case that microwaves themselves, not heat, is the cause for the destruction of the enzyme. Note that when you have a conclusion that boosts one idea over another, in some ways it becomes a two-part conclusion. Here the two parts are:

  1. Heat does not cause the destruction of the enzyme.

  2. Microwaves themselves do cause the destruction of the enzyme.

In order to weaken this conclusion, we could really have something that attacks either part. For example, if we had a different kind of heater which was neither a microwave nor a conventional system that also destroyed the enzyme, that could at least partially call into question the claim that microwaves themselves are the issue.

In this case, however, answer choice (E) is going to attack point 1. It will make the case that the cause of the breakdown of the enzyme really is heat, however, heat is administered differently in a microwave versus a conventional heater.

That's where the idea of those different pockets of superheated liquid come in. If this is how microwaves typically heat a liquid, it could be that the heat is destroying the enzyme in the extra hot pockets of liquid. Yes, microwaves are still to blame, but in this case it is how microwaves transmit heat, not microwaves instead of heat.

PrepTests ·
PT134.S3.Q24
User Avatar
Benjamin Kieta
Wednesday, Jun 04

Student question:

When the stimulus is short, can we consider that if then statements to always be the conclusion? The first time I saw this stimulus I thought if then provided support for the second sentence.

Tutor answer:

No, unfortunately we can't always rely on an if-then statement to be the conclusion, even in a shorter stimulus. In fact, it's more common for an if-then statement to be a premise rather than a conclusion.

This stimulus is a pretty tricky one for deciding which statement is the premise and which is the conclusion. Unfortunately, we don't have and decisive indicator words that tell us 100% which is which.

When that happens, we have to use judgment to try to determine which is better understood as the support for the other. One way you can do this is by asking the question "why". Whichever statement better answers the "why" question for the other is the premise. In this case, it makes more sense to see the second claim as answering the question "why" for the first claim.

Confirm action

Are you sure?